LSAT and Law School Admissions Forum

Get expert LSAT preparation and law school admissions advice from PowerScore Test Preparation.

 LSATer
  • Posts: 47
  • Joined: Nov 13, 2016
|
#33344
This is a justify question stem, correct?

Thank you,

LSATer
 Steven Palmer
PowerScore Staff
  • PowerScore Staff
  • Posts: 35
  • Joined: Feb 21, 2017
|
#33345
Hi LSATer,

That's right! #23 asks, "Which one of the following, if assumed by the author of the passage, would allow her properly to draw her conclusion...".

We know it's a justify question mainly because of the "properly draw her conclusion" part.

Best,

Steven
 LSATer
  • Posts: 47
  • Joined: Nov 13, 2016
|
#33349
Thank you! I got it wrong the first time, because I approached it as a MBT Inference question (and chose B) but then I looked at it again as a Justify question and I got it right.
 Anali
  • Posts: 18
  • Joined: Apr 01, 2017
|
#40169
Hello Powerscore Staff!

Thank you for taking time to respond to my inquiries. I have read your explanations and have found them extremely helpful. I am treading forward in the QT training books. I have another inquiry; this one is regarding PT14-S4-Q23: "A car's antitheft alarm that sounds..."

I want to see if my thought process was correct in how I eliminated answer choices.

(A) does not support the conclusion that people should deactivate their cars' antitheft alarms when they park them in crowded city neighborhoods at night. This ac actually seems to weaken the conclusion.

(B) "In most cases" raises a red flag, because it is weak wording. Is it safe to assume that we should be weary of weak wording in the answer choices for a Justify question? This ac leaves room for those instances during which a car's alarm will go off because it is being broken into--this would weaken the conclusion as well.

(C) is correct because it provides a reason why antitheft alarms should be deactivated: undisturbed sleep is more important. When reading through the stimulus, however, I was unable to ascertain how sleep is more important than preventing car theft. I understand that justify answer choices can be superfluous, but this seems to be the reason why this ac is correct. :-?

(D) this ac is similar to (B) in that it entails weak wording and leaves room for the possibility that people with antitheft alarms can also be considerate of others: the alarm can scare carjackers away from the neighborhood.

(E) can be eliminated because we are concerned about what happens at night, not during the daytime.


Thank you,


Anali
 Jennifer Janowsky
PowerScore Staff
  • PowerScore Staff
  • Posts: 90
  • Joined: Aug 20, 2017
|
#40181
Hi Anali!

You are actually correct that the answer was (C), and your thought process for eliminating the others was also correct. Test makers like to make the right answers seem unlikely. For Justify Questions especially, this can be done by saying something strange like "undisturbed sleep is more important than preventing car theft." This doesn't have to be true, nor does it have to be proven by the stimulus information, it only needs to support the stimulus, which in this case it does. :)
 presleys
  • Posts: 3
  • Joined: Feb 22, 2024
|
#105570
I got this question right and it makes sense to me that C is the strongest answer. However, I am still having a hard time understanding what makes B incorrect. If we use the justify formula then, "the sleep of many people in the neighborhood is disturbed" + "In most cases when a car alarm sounds at night, it is a false alarm." therefore "out of consideration for others, people who have these antitheft alarms on their cars should deactivate them when they park in crowded city neighborhoods at night." makes sense to me. I suppose if we use the mechanistic approach, disturbed sleep appears in the premises and not the conclusion and therefore should appear in the correct answer, but then again false alarms appear in the premises and not the conclusion as well. Can someone clear this up for me?
User avatar
 Hanin Abu Amara
PowerScore Staff
  • PowerScore Staff
  • Posts: 60
  • Joined: Mar 29, 2023
|
#105583
Hi Presleys,

Great question. In Justify we are essentially strengthening to the 100%. The problem with B is the word "most cases." While most cases is compelling and does strengthen the argument, it doesn't completely prove the argument. Because if out of 100 cars, 60 are false alarms and 40 aren't, that might be a good enough reason to not turn off your alarm.

Hope that helps.

Get the most out of your LSAT Prep Plus subscription.

Analyze and track your performance with our Testing and Analytics Package.